LSAT and Law School Admissions Forum

Get expert LSAT preparation and law school admissions advice from PowerScore Test Preparation.

 jared.xu
  • Posts: 65
  • Joined: Oct 07, 2011
|
#2213
This is a must be false question. The stimulus to this problem is not so difficult to understand, but the answer choices are confusing. A, D, and E is in the form of either/or. Is A the right answer because both parts of "either" and "or" are wrong, whereas D is "could be true" because although the first part (either) contradicts the stimulus, the second part (or) is possible? Is E also "could be true" because although the first part (either) contradicts the stimulus, the second part (or) is "must be true"? Is my analysis correct that for an "either/or" answer choice to be "must be false," we need both parts (the "either" part and the "or" part) contradicting the stimulus?

And then why is C not the right answer? C contradicts the stimulus because it states that the economy is weak "only if investment decreases," whereas the stimulus clearly allows more than one necessary condition, such as prices remaining constant and unemployment rising. Thank you in advance for replying.
 Nikki Siclunov
PowerScore Staff
  • PowerScore Staff
  • Posts: 1362
  • Joined: Aug 02, 2011
|
#2254
The stimulus can be diagrammed as follows:

Premise 1: Weak Economy --> Prices Constant and Unemployment Rises

Premise 2: Unemployment Rises --> Investment Decreases

Premise 3: NO Investment Decrease

Since there is NO investment decrease, we can conclude that unemployment is NOT rising, and as a result, the economy is NOT weak.

Answer choice (A) must be false, because both claims in the either/or statement are not true. The economy is NOT weak, and investment is NOT decreasing.

Answer choice (C) contains a Must Be True statement:

Weak Economy --> Investment Decrease

Although investment decreasing is not the only necessary condition for the economy to be weak, it is certainly one of several necessary conditions based on the stimulus.

Answer choice (D) could be true, because the prices could remain constant even if the economy is not weak: we just don't know if they are or aren't. To conclude otherwise would be to make a Mistaken Negation. So, although we do know that the economy is NOT weak, it is possible that prices are remaining constant.
 jared.xu
  • Posts: 65
  • Joined: Oct 07, 2011
|
#2345
Thank you for the explanation. I guess I misinterpreted "only if" in "C The economy is weak only if investment decreases." "Only if" does not imply that the investment decreasing is the only necessary condition, and no other could be the necessary condition, correct? And is it true that for an "either/or" statement in this case, if one part of "either" phrase or "or" phrase is "could be true" or "must be true" regardless of the other part, the answer should be eliminated? In E, even though the "either" phrase is "could not be true," the "or" phrase is "must be true." And E should be eliminated because of the "or" phrase, correct?
 Nikki Siclunov
PowerScore Staff
  • PowerScore Staff
  • Posts: 1362
  • Joined: Aug 02, 2011
|
#2432
Correct. "Only if" modifies a necessary condition, but it does not mean that the condition modified is the only necessary condition: you will go to Harvard Law only if you ace the LSAT. Acing the LSAT is a necessary condition, but clearly there are other conditions necessary to attend Harvard.

(E) is incorrect because the second part of the "either/or" statement must be true. For an answer to be correct, given the "cannot be true" question stem, both parts of the answer must be false (i.e. cannot be true).
 moshei24
  • Posts: 465
  • Joined: Mar 20, 2012
|
#5114
Hi!

I looked at this question for a really long time two days after having it in the experimental section on a practice test, and I came to a conclusion, though, I'm not sure if my conclusion is accurate.

The way I see the question now is as such:

Assumptions/Premises:
A1: Economy Weak --> Prices Remain Constant + Unemployment Rises
A2: Unemployment Rises --> Investment Decreases
A3: Currently, investment is not decreasing

Inferences:
I1: Economy Weak --> Investment Decreases
I2: Investment *not* Decreasing --> Unemployment *not* Rising --> Economy *not* Weak (the tricky thing here is that it doesn't affect whether or not the prices remain constant).

(A) must be false since the current state is that investment is not decreasing, the economy CANNOT be weak, and investment CANNOT be decreasing unless something else happens to modify its current state.

I'm not sure if I analyzed that correctly, because I'm not sure how the last assumption plays in, and I'm also confused by the wording A or B. When it uses "or" in an answer choice in a must be false question, how is that wording deciphered?

A clarification of this question and of how to decipher the uncommon terminology used in this question would be greatly appreciated.

Thanks!
User avatar
 Dave Killoran
PowerScore Staff
  • PowerScore Staff
  • Posts: 5850
  • Joined: Mar 25, 2011
|
#5152
moshei24 wrote:The way I see the question now is as such:

Assumptions/Premises:
A1: Economy Weak --> Prices Remain Constant + Unemployment Rises
A2: Unemployment Rises --> Investment Decreases
A3: Currently, investment is not decreasing
This is all correct, although since they are stated explicitly, they are premises and not assumptions. That point is minor, though, and not material to the validity of the analysis.
moshei24 wrote:Inferences:
I1: Economy Weak --> Investment Decreases
I2: Investment *not* Decreasing --> Unemployment *not* Rising --> Economy *not* Weak (the tricky thing here is that it doesn't affect whether or not the prices remain constant).
This is also correct. We could also say there is a contrapositive of A1 above, but I'm certain you rightly left that off since it is inherently true and thus not a true "inference." You also noted the pricing issue uncertainty, which is an intentional part of the construction of the problem (meaning they have that there on purpose, as it adds confusion to the problem).
moshei24 wrote:(A) must be false since the current state is that investment is not decreasing, the economy CANNOT be weak, and investment CANNOT be decreasing unless something else happens to modify its current state.
Again, you are correct.

moshei24 wrote:I'm not sure if I analyzed that correctly, because I'm not sure how the last assumption plays in, and I'm also confused by the wording A or B. When it uses "or" in an answer choice in a must be false question, how is that wording deciphered?

A clarification of this question and of how to decipher the uncommon terminology used in this question would be greatly appreciated.

Thanks!
The last "assumption" is just a stated fact, and one that ultimately enacts a contrapositive chain.

The interesting part of this problem is the wording of the answers. (A), (D), and (E) each feature an "either/or" construction, so let's take a moment to consider what that means. When "either/or" is used as it is here, the meaning is "at least one." Thus, each of these answers is saying, "At least one of these two things occurs." If an answer cannot be true, then it must be the opposite of that, or, in other words, that neither can occur. That is what occurs in (A), and why answer choice (A) is correct.

Please let me know if that helps. Thanks!
 moshei24
  • Posts: 465
  • Joined: Mar 20, 2012
|
#5154
Thank you. That makes it clear.

Just to make sure I understand it clearly, I'm going to re-say what you said in my own words.

When you have "or" in a question choice, it means that one of the two has to be true, so in a must be false question, if neither one can be true, the answer must be false.

Let's say it was a must be true question. How would the "or" construct work in that type of question? If either one is true, then that choice is the correct answer choice because one of those two answers must be true? Because it must be true that A or B is true? (A referring to one thing and B another thing, not referring to answer choices.)

Thank you!
 moshei24
  • Posts: 465
  • Joined: Mar 20, 2012
|
#5161
Thanks.
 RayMiller
  • Posts: 33
  • Joined: Jul 16, 2012
|
#6901
I'm having trouble being sure that I am correctly explaining to myself why the incorrect answers are not correct and questioned the way I diagrammed the prices constant and the umemp rise. This is how I diagrammed.

Stimulus

Economy Weak :arrow: Prices constant and Unemp Rise
Umemp Rise :arrow: Invest decreases
Linked:
Economy Weak :arrow: Prices constant and Umemp Rise :arrow: Invest decreases

Contrapositve:
Prices not constant or Unemp does not rise :arrow: Economy not weak
Invest does not decrease :arrow: Umemp does not rise
Linked:
Invest does not decrease :arrow: Umep does not rise or prices not constant :arrow: economy not week

Conclusion: Invest does not decrease, so....this is what I know must be true:
Invest does not decrease :arrow: Umep does not rise or prices not constant :arrow: economy not week

Question Stem: What cannot be true.

A. Economy not weak :arrow: Investment decreasing (Correct Answer- Can't Be True)
B. Umemp Rise :arrow: Prices Constant (Could Be True?)
C. Economy weak :arrow: Investment decreases (Must Be True)
D. Economy not weak :arrow: Prices constant (Could Be True b/c of or statement)
E. Umemp not Rise :arrow: Economy not weak (Must Be True)

Thanks so much for your help

Get the most out of your LSAT Prep Plus subscription.

Analyze and track your performance with our Testing and Analytics Package.